How to Find an Upper Bound for Derivative Error in Lagrange Formula

Click For Summary
The discussion focuses on finding the nth derivative of the function f(t) = ln(1 + 2x) and establishing an upper bound for the derivative error using the Lagrange error bound. The nth derivative is given as f^(n)(x) = (-1)^(n+1) * 2^n / (n * n!) * (1 + 2x)^(-n). Participants note that while the Lagrange error bound is typically used for Taylor polynomials, it can also apply to the derivative generator. To find the maximum value M for the error bound, one must analyze the behavior of the derivative on the specified interval of -0.25 to 0.25. The conversation emphasizes the importance of estimating the maximum derivative to effectively apply the error bound formula.
brunie
Messages
62
Reaction score
0
Consider the function f(t) = ln (1 +2x)

Give a formula for f^(n) (x) [**the nth derivative] valid for all n >= 1 and find an upper bound for | f^(n) (x) | on the interval -0.25 <= x <= 0.25.
[ the error ].

I found the nth derivative to be

f^(n) (x) = (-1)^n+1 * 2^n /n * n!
--------------------------------
(1 + 2x)^n

so for
first derivative = 2 / 1+2x
second " " = -4 / (1+2x)^2
third " " = 16 / (1+2x)^3
etc.

now for the error i kno there is a lagrange error bound equation for taylor polynomials, but the question isn't for the taylor polynomial, only the "derivative generator"

i kno the max |f^(n+1)| <= M on an interval

so i just need help dealing with only the derivative error and i also want to kno how to find the M value in general with Taylor polynomials (not part of the above question)

where |f(x) - P_n(x)| <= M / (n+1)! * |x-a|^(n+1)
for interval between a and x
 
Physics news on Phys.org
You are essentially done when you minimize the denominator in absolute value. Note, only "an upper bound" is asked for: You have room to over estimate.
 
Question: A clock's minute hand has length 4 and its hour hand has length 3. What is the distance between the tips at the moment when it is increasing most rapidly?(Putnam Exam Question) Answer: Making assumption that both the hands moves at constant angular velocities, the answer is ## \sqrt{7} .## But don't you think this assumption is somewhat doubtful and wrong?

Similar threads

  • · Replies 6 ·
Replies
6
Views
2K
  • · Replies 8 ·
Replies
8
Views
2K
Replies
3
Views
2K
Replies
1
Views
3K
  • · Replies 6 ·
Replies
6
Views
4K
  • · Replies 23 ·
Replies
23
Views
2K
Replies
6
Views
3K
  • · Replies 6 ·
Replies
6
Views
1K
  • · Replies 27 ·
Replies
27
Views
3K
  • · Replies 1 ·
Replies
1
Views
2K